You are on page 1of 25

SOLUTIONS FOR DATA INTERPRETATION REPLICA QUESTIONS THAT HAVE APPEARED IN CAT IN THE LAST 4 YEARS

TABLES Solutions for questions 1 to 3: 1. Let the volume of data transfer in India and Singapore be 100 units each. Revenue from data transfer in India = 100 1 = $100 Revenue from data transfer in Singapore = 100 9 = $900 100 Total revenue in India = 100 = $1111 9 100 Total revenue in Singapore = 900 = $4285 21 Total revenue in Singapore is about 4 times that in India. Choice (5)

Average gross pay =

108800 = 18133 6 18133 16000 percentage increases = 100 16000 13% Choice (3)

8.

2.

Revenue from data transfer as a percentage of total revenue for India in 2010 = 27% Revenue from data transfer as a percentage of total revenue for Sweden in 2010 = 36% Let total revenue in India in 2010 be $200 and that in Sweden be $100 ARDT of Sweden = $6 36 Volume of data transfer in Sweden = =6 6 Volume of data transfer in India = 6 54 ARDT in India = =9 6 9 1 The percentage increase = 100 = 800% 1 Choice (3) It can be seen that if the total revenue received is the same for the given pairs of countries, only UK and Spain would have approximately the same volume of data transfer. Choice (4)

As after the mutual transfer, the average age of the Finance department increase by one, it means that the age of the person who came from the Marketing department was 20 years older than the age of the person who was transferred from the Finance department. Now after the transfer of the employee to the HR department, as the average age of the employees left in the Marketing department remained the same, the age of the employee transferred to the HR department, was 20 years younger than the average age, i.e., 36 20 = 16 years. The new average age of the employees in the HR department 46 5 + 16 1 246 = Choice (3) = = 41 years 6 6 The new average basic pay of employees in the HR 10,000 5 + 12,000 2 + 16,000 1 department = 8 90000 50,000 + 24,000 + 16,000 = = = 11250 8 8 The percentage change = 12.5% Choice (2)

9.

Solutions for questions 10 to 13: 10. The drink must contain 10% minerals. As there are only two drinks (A and C) with 10% minerals, the drink can be prepared in only one way. As A and C have 30% protein each, they can be mixed to form the drink. Choice (1) 11. None of the choices (1), (2) and (3) can be used to form the drink with 10% fat and at least 30% protein. For C and E to form the drink with 10% fat and at least 30% protein, if they are mixed in the ratio x : y (say) x (50 )+ (0 ) = 10, x : y = 1 : 4 x+y

3.

Solutions for questions 4 to 6: 4.

To get calls from all the colleges, Arun should have scored at least the highest value of cut-off in each section, i.e., 44 + 44 + 45 + 44 = 177 and also at least the highest value of aggregate cut-off for any institute, i.e., 176. Choice (2) The minimum aggregate marks to get calls from two colleges is 171. If he scores 50 each in three sections he needs to score at least 21 marks in the fourth section. Choice (3) Four colleges have a cut-off for section C and the remaining two colleges have a cut-off for section D. If a student misses the cut-off in these two sections, he/she would miss calls from all the colleges. The maximum possible marks such a student gets is 50 + 50 + 40 + 42 = 182. Choice (3)

5.

6.

1(200 )+ 4 (100 ) 600 = 120 = 5 5 Similarly the ratio for D and E is 1 : 3 and the cost per 800 unit is = 200 4 The cost per unit is the least for C and E. Choice (4) cost per unit =

Solutions for questions 7 to 9: 7.

The new gross pay of the employee transferred 80 = 16,000 + 16000 = 16,000 + 12,800 = 28,800 100 The gross pay of the current employees in HR department = 16000 5 = 80000 New gross pay of the six employees = 80,000 + 28,800 = 1,08,800

12. The drink should have at least 60% carbohydrate. Further in the mixture formed by B, C and E, the proportion of B should be maximum and the other two should be minimum to get the lowest per unit cost. Among the given options only Choice (2) and (5) satisfy the condition having 60% carbohydrate and of these, choice (5) has the lowest per unit cost. Choice (5) 13. A and B when mixed in equal proportions, the protein 30 + 20 content will be only = 25%, which is less than 2 required. D and E when mixed in equal proportion, the

carbohydrate content will be only

5 + 45 = 25% which is 2 less than required. Similarly B and E and C and D when mixed in equal proportion the combination will have less than the required percentage of minerals and carbohydrate respectively. Only A and E when mixed in equal proportion would yield a mixture with all the contents in the required amount. Choice (5)

Solutions for questions 18 to 21: 18. The costs of a refrigerator, an air conditioner and a music system in different countries are.

(00 U.S. dollars) India Thailand Malaysia Singapore USA 11 + 5 13 + 5 11 + 6 13 + 4 20 9+7 12 + 5 10 + 8 12 + 5 23 8.5 + 9 10 + 6 8+4 13 + 4 20 49.5 51 47 51 63 Choice (3)

Solutions for questions 14 to 17: 14. If one observes the values given for the different parameters, the values that were varying with production, i.e., value was increasing when production increased and value decreasing when production decreased are material, labour and operating cost of machines. All the remaining costs, i.e., rent of building, consumables, rates and taxes, repair and maintenance expense and selling and marketing expenses are fixed. Hence, there will be no change in these costs. The total fixed cost = 1800 + 600 + 1200 + 8700 + 2100 = 14400 The cost/unit for different variable costs is as follows. Material = `50 per unit. Labour = `20 per unit Operating cost of machine = `30 per unit Total = `100 per unit Selling price per unit = `125 per unit 14400 Total cost/unit for 2100 units is `100 + 2100 = `107 Choice (2) 15. For one product, Selling price = `125 Variable cost = `100 _________________ Difference = `25 _________________ Now, to avoid loss, the company has to offset the fixed cost (i.e., 14400) for which it has to produce a total of 14400 = 576 units. Choice (3) 25 16. The reduction in selling price per unit = 5% of 125 = 6.25 New selling price = 118.75 Total fixed costs = `14400 Variable cost per unit = `100 Now the total profit increases with the increase in number of units sold and the maximum profit is obtained when the company sells and 3000 units. Choice (5) 17. The given condition is that if the company sells upto 2100 units, the selling price per unit is `125 and if the company sells 2550 units, the selling price per unit for all the units is `120. The profit of the company increases upto a production figure of 2100 units, from the 2100th unit to the 2101st unit, the total profit decreases drastically and from the 2101st unit to the 2550th unit, the profit again increases.

Refrigerator Air conditioner Music system Total

The cheapest is in Malaysia.

19. As given in the previous question, the total cost will be highest in India (850 + 900 = 1750) Choice (1) 20. Cost in India = 300 + 500 = 800 Cost in Thailand =450 + 600 = 1050 Difference = 250 32.9 = 8225 Duty = 1500 Required difference = 6725

Choice (4)

21. Cost in India with dollar at `40.92 = 550 40.92 2500 22500 Cost in India with dollar at 35 = = 650 35 Cost in Singapore = 900 Required difference = 250 Choice (2) Solutions for questions 22 to 26: 22. Let us check the possible short routes from A to J.

Total cost
Rs. 335 Rs.1135 A B J 280 1150 km

Total distance 1430 km 1250 km 1155 km 1090 km 1175 km

`1470 `1850 `1425 `1670 `1135

Rs. 625 Rs.1225 D J Rs. 25 km 825 km

A A A A

Rs. 850 Rs. 575 F J 670 km 485 km Rs.1225 Rs. 445 G J 675 km 485 km Rs. 925 Rs. 210 H J 975 km 200 km Rs. 675 Rs. 215 C 395 km 205 km

Rs. 575 J 485 km

`1465

1085 km

The shortest possible route is A C F J. The cost is `1465. Choice (4)


23. The route with the least cost is A H J, with a total cost of `1135. As the cost of the new service is 5% less then `1135, it should be 1135 (5% of 1135) = 1078. Choice (2) 24. If C, D and H are closed, then the minimum cost of Choice (3) travel is for A F J, i.e., `1425. 25. We want the
Pr ice to be as minimum as possible. dis tan ce It is less than 1 in only the cases A H, B J and

Hence, the profit would be maximum at the production figure of 2100 units or at 2550 units. Production Selling price / unit (s) Variable cost / unit (v) SV (S V) production Total fixed cost Total profit 2100 units `125 `100 `25 25 2100 = 52500 14400 38100 2550 units `120 `100 `20 20 2550 = 51000 14400 36600 Choice (1)

The maximum profit is `38100

C D. Considering the cases involving the above routes. Route AHJ ABJ Price / Distance 1135 1175 1470 1430 Taking margin of 10% into Account 1135 10 1175 11 1470 10 1430 11
1135 10 1175 11

27. Percentage of male employees in the production 288 100 = 45 Choice (2) department = 640 28. Post graduates in the marketing department = 32 25 32 = 8 Male postgraduates = 100 Female post graduates = 32 8 = 24 Male non post graduates = 48 8 = 40 Required difference = 40 24 = 16 Choice (5) 29. Percentage of male post graduates in the marketing 32 department = 100 = 40 Choice (1) 80 30. The number of male post graduates in the production department = 144. Female post graduates = 352 144 = 208 The number of male and female post graduates and male and female employees who are not post graduates are as follows.

It will be the least for A H J and is = 103.2 = .88 117.5

Choice (2)

26. The cost / kilometer is the least for A H J and the distance is 1175 km. Choice (4) Solutions for questions 27 to 30:

With the given information we can deduce the number of males and postgraduates in the different departments as follows: Department Marketing Accounts Production Total Total 80 80 640 800 Male 48 44 288 380 Post graduates 32 40 352 424

Post graduate Male 144 Females 208

Non Post graduates Males 144 Females 144

It can be seen that except female post graduates all other groups (male posts graduates, male and female non post graduates) have the same number of employees. Choice (3)

Solutions for questions 31 to 33: 31. The total number of bookings made is the highest in Q3 and so the average number of bookings per month is also the highest. Choice (C)

Month Number of bookings Number of deliveries

Jan 346

Feb 412 200

March 380 216 (146)

April 450 160 (196)

May 308 225 (220)

June 359 170 (225)

July 462 159 (138)

Aug 333 296 (200)

Sep 345 134 (166)

Oct 250 50 (199)

Nov 506 125 (295)

Dec 370 278 (125)

32. The values shown in the brackets are of the booking made 2 months ago. Number of deliveries made in August from the bookings made in June = 200. Number of deliveries made in December from the bookings made in November = 278 278 Choice (A) = 1.39. 200 33. We only need to check the revenue for quarters Q3 and Q4. Revenue (in `) from Q3 = (462 + 333 + 345) 43,100 = 4,91,34,000 Revenue (in `) from Q4 = (250 + 506 + 370) 44,000 = 4,95,44,000 the highest revenue is obtained from Q4 i.e., `4,95,44,000 Choice (B) Solutions for questions 34 to 36: 34. The given condition occurs in the case where the number of wins is in the range 16 18. Choice (B) 35. From the given table, the 3 least percentage occurs in the 99 94 100 100 = last row i.e., for 3133, which is 95 19 5 Choice (C) = 5 % 9
rd

36. The given condition is satisfied in the case, where the number of wins is in the range 25 27 i.e., 68 + 64 + 63 Choice (A) = 65 = 2.5 26 3 Solutions for questions 37 to 39: 37. Investment (in `) in NLP Industries before withdrawal = 12.5% (12,00,000) = 1,50,000 Investment (in `) in NLP Industries after withdrawal = 16% (9,00,000) = 1,44,000 the percentage change in investment 1,50,000 1,44,000 = Choice (D) 100 = 4% 1,50,000 38. The return on investment for Mr. Anil 2 25 = 7,00,000 = 3500 100 100 The return on investment for Ms. Shivani 2.5 10 = 13,00,000 = 3250 100 100 Therefore the required difference = (3500 3250) Choice (A) = `250

39. The three persons A, B, and C made an investment of `10 lakh, `20 lakh and `21 lakh respectively such that their investments fall under the schemes X, Y, Z respectively.

Their combined return on investment (in ` ) = 2% (10,00,000) + 2.5% (20,00,000) + 3% (21,00,000) = 20,000 + 50,000 + 63,000 = 1,33,000 Their combined return on investment after the firm increased the rate of return = 2.2% (10,00,000) + 3% (20,00,000) + 3.3% (21,00,000) = 22,000 + 60,000 + 69,300 = 1,51,300 The required increase (in `) = 1,51,300 1,33,000 = 18,300 Choice (D)
Solutions for questions 40 and 41: 40. Percentage contribution of mono speaker of the 1000 100 = 12.3% company NOSY in 2001 = 8100 Percentage contribution of mono speaker of the 1600 100 = 17% company BOSS in 2001 = 9400 Percentage contribution of mono speaker of the companies NOSY and BOSS in 2003 are 13.5% and 19.6% respectively. the percentage contribution of mono speakers of both the companies increased. Proceeding, similarly we observe that for no other type of music systems of both the companies, the percentage contribution increases. Choice (B) 41. Percentage contribution in 2001 12.3% 22.22% 28.4% 17.3% 19.75% Percentage contribution in 2003 13.54% 22.9% 18.75% 23.95% 20.8

The franchise in Bengaluru will earn more revenue then the establishment fees (in each of the two centres) after one year. Choice (D)
44. If a customer spends on an average `300 and `130 at a Foodie restaurant in class A center and class B center respectively, then the total number of customers who are required to come such the revenues are not less than the establishment fees would be the i.e.,
132 10 5 104 10 5 + = 44,000 + 80,000 = 1,24,000 300 130 Choice (B)

Solutions for questions 45 and 46: 45. As no information is given regarding the percentage of dropouts for districts R and S in few years, (A) cannot be definitely a false statement. As no information is given about the number of enrolments in each districts in any of the years, statements (B) and (D) cannot be confirmed. (C) is definitely false because the dropout percentage of district Q in any of the given years is greater than that of each of the other districts. So, the overall dropout percentage would also be the highest. Choice (C) 46.

Type of music system

Mono speaker Dual speaker 1000w Dual speaker 2000w Four speakers 5000w Home theatre

District P Q R S T

Minimum number of achievements 5 1 2 0 2

total number of achievements (minimum = 5 + 1 + 2 + 0 + 2 = 10) Choice (B)


Solutions for questions 47 and 48: 47. The rise in temperature (in C) per hour =

The maximum change in percentage points occurs for Dual speaker 2000W. Choice (C)
Solution for question 42: 42. The total number of cars sold by showroom A and showroom B at the end of 7 days are 209 and 221 respectively. 209 = 100 94.5%. Hence statement is true. 221 The total number of cars sold by showrooms on odd numbered days = 16 + 35 + 33 + 51 + 60 = 195 The total number of cars sold by showroom B on even numbered days = 19 + 42 + 29 + 52 + 81 = 223. 90%(223) = 200.7 Statement is also true. Choice (C) Solutions for questions 43 and 44: 43. Hyderabad:
126 10 5 = 52,500 > 51,860; 240

46 25 =7 7

temperature (in C) in city Q at 10 a.m. = 3 (10 5) + 25 = 40 Choice (D)


48. Temperature in city P at 3.30 p.m. 1 42 29 = 42 3 6 2

= 42

91 34.5C 12
City Q R S T U Temperature at 3.30 p.m. 35.5 C 37 C 38.5 C 36.66 C 33.5 C

Similarly,

75 10 5 < 42,500 180 a restaurant in class B center but not class A center will earn more revenue then the establishment fee in one year.

city S has the highest temperature at 3.30 p.m. Choice (C)


Solutions for questions 49 to 52:

Bengaluru:

144 10 240

= 60,000 < 60,200

P Q R S T U Traffic flowing from 3,346 3,752 2,536 2,620 2,952 3,060 Traffic flowing to 3,504 2,612 3,308 2,852 3,050 2,940
49. Total traffic through the route P Q = 964 + 846 = 1810. Similarly verifying it is easy to see that the maximum traffic flow occurs through the road connecting PQ. Choice (A)

90 10 5 = 50,000 < 50,246 180

50. Looking at the table and relating the diagonal elements, it is easy to see that the 2nd least traffic flow occurs through the road connecting Q S. Choice (D) 51. From the above table, traffic flowing from city Q is the greatest i.e., 3752 vehicles. Choice (B) 52. From the above table, the difference in traffic flow is the least for city T i.e., 3050 2952 = 98. Choice (C) Solutions for questions 53 and 54: 53. Interest amount for Mr. A (in `) = 3,600 2.2 = 7,920 Interest amount for Mr. B (in `) = 3,800 3.6 = 13,680 The required difference (in `) = 13,680 7,920 = 5,760 Choice (B) 54. The required average =

Solutions for questions 59 and 60: 59. The gain from the shares of company IV in 2006 was = [ 448 + 432 2( 456)] 132 + 2 = 132 16 = 116 Choice (1) 60. We can only evaluate the return from the shares of company III in the years 2002 to 2009. The returns were as follows:

Year Gain

2002 2003 2004 2005 2006 2007 2008 2009 122.5 158.5 155 150 170 148 172 195

The highest percentage increase is from 2002 to 2003 and it is 29.38% Choice (2)
Solutions for question 61: 61. To find the median, arrange the per capita incomes in descending (or ascending) or order.

1 [5.6 4,800 + 6.4 4,000] 2 1 = [26,880 + 25,600] 2 Choice (C) = `26,240.

Solutions for questions 55 and 56: 55. Let the total number of employees in company X, company Y and company Z be x, y and z respectively. Male employees in company x who owns both four wheeler and two wheeler = 0.7x 0.15 (Q 45 + 65 + 5 100 = 15) = 105x Female employees in company y who owns both four wheeler and two wheeler = 0.3x 0.1 = 0.03x. The total number of employees in company X who owns both four wheeler and two wheeler = (0.105 + 0.03)x = 0.135x percentage of employees = 13.5% Similarly for company Y, the required percentage is 26% Similarly for company Z, the required percentage is 6%. Choice (B) 56. Let the number of employees in either of the companies be n. The number of male employees in companies Y who satisfy 1 [100 (30 + 20)]n = 0.5n the given condition = 100 similarly the required number of employees in company

Per capita income ($) 24,369 24,337 23,484 19,207 15,350 13,746 13,477 11,692 10,372 5,663 4,965 3,523 2,916

Country Switzerland Germany United states United kingdom New Zealand Swedes France Spain Hong Kong Brazil Latherier Mexico Romaine

Z=

1 [100 (20 + 10)]n = 0.7n. 100

Out of the 13 countries, the median is the country placed 7th. France with a per -capita income of $13,477. 40% of 13,477 = $ 5390.8. There are 10 whose per capita income is more than $5390.8. Choice (2)
Solutions for question 62: 62. Given, intra-state services accounted for 60% of total revenues. 2880 Total revenues = = `4800 crore. 0.60 Total revenues from non A/C general category in intra-services is given to be 50% of revenues from intrastate services. Revenues from non A/C general category in intrastate services = 50% of 2880 = `1440 crore Choice (2) Solutions for question 63: 63. The total production of the top four coal producing countries is 2536.7 + 1039.2 + 478.2 + 393.9 = 4448 mt The total production of the bottom four coal producing countries is 76.7 + 76.6 + 145.8 + 174.9 = 474 mt. 474 The required percentage = 100 = 10.66% 4448 Choice (1)

the required percentage =

0. 5 n + 0. 7 n 100 = 60%. 2n Choice (D)

Solutions for questions 57 and 58: Model RL-100 BCZ Thunder WB-150 Muzzle No. of Bikes Sold Average selling price (in `) 2007 40,000 25,000 31,000 20,000 52,000 2008

19,500 37,500 30,000 45,000 18,000

45,000 28,000 35,000 23,000 55,000

57. From the above table, the percentage increase in the average selling price is the highest for WB-150. 23,000 20,000 100 = 15% Choice (C) 20,000 58. The required average (in `) 40 + 25 + 31 + 20 + 52 = 103 5 168 = 10 3 = 33,600 5

Choice (B)

Solutions for questions 64 and 65: 64. Male students who were eligible for selection were A, F, G and N and the female students who were eligible for selection were L and O Therefore the required ratio is 2 : 1 Choice (1) 65. x = 6 and y = 4 There fore 2x = 3y is in the correct choice. Choice (4) Solutions for question 66 and 67:

The Total costs, Operating Expenses, Revenue and the Profitability of the company in the five years are given in the following table. Year 2009 2008 2007 2006 2005 Total cost (in `) 92200 83700 89600 96600 104000 Operating Revenue Profitability Expense (in `) (in `) (in `) 24050 104200 0.2308 21775 96600 0.2254 22000 112400 0.1957 24730 128200 0.1929 26580 130600 0.2035

71. The growth rate in imports of the 4 companies from 2002 03 to 2003 04 are as follows: 6.53 Rahual & co: 100 = 127% 5.14 4 .7 Chandu & co: 100 = 40.5% 11.61 0.67 Shiva & co: 100 = 7.7% 8.72 4.05 Kanta & co: 100 = 54.3% 7.46 Hence the growth rate of imports is the least for Shiva & co. Choice (3) 72. Trade deficit = imports exports The trade deficit of the 4 companies in 2004 05 are as follows Rahul & co: 2.11 Shiva & co: 0.9 Chandu & co: 1.07 Kanta & co: 0.6 Rahul and co has the highest trade deficit in 2004 2005 Choice (1) Solutions for questions 73 and 74: 73. The number of employees who did not cross the cut off for all the 5 companies are as follows.

66. The profitability of the company was the least in the year 2006 Choice (2) 67. With respect to the previous year in the years 2006, 2007 and 2008 were 7.11%, 7.24% and 6.58% respectively. In 2009 the total cost increased when compared to 2008. Therefore the maximum decrease was in 2007 and it was 7.24%. Choice (2) Solutions for questions 68 and 69:

Sections A B C D E Total

Score Score less Score from Total no of greater than than 45 45 to 85 students 85 28 72 24 124 15 68 36 119 18 52 28 98 29 58 47 134 30 60 35 125 120 310 170 600

A B C D E

No. of employees who did not cross the cut off. 120 225 100 200 275

Hence E has rejected the maximum number of employees. Choice (4)


74. The number of employees who got more than 90% for the 5 companies are as follows:

68. Percentage of the total number of students getting 120 scores less than 45 = 100 = 20% Choice (4) 600 69. For the sections A, B, C, D and E, the maximum number of students getting 48 or more in the examination was 96, 104, 80, 105 and 95 respectively. Thus the highest among the above values is 105. Therefore the maximum number of students from a section who passed in the examination was 105. Choice (3) Solutions for questions 70 to 72: 70. The growth in exports of the 4 companies from 2003 04 to 2004 - 05 are as follows.. 3.15 Rahual & co: 100 = 25.9% 12.15 2 .1 Chandu & co: 100 = 14.9% 14.1 2 .8 Shiva & co: 100 = 30.1% 9 .3 3.32 Kanta & co: 100 = 31.9% 10.41 Hence Kanta & co has the highest growth in exports. Choice (4)

A B C D E Total

Greater than 90% marks 30 36 30 96 115 275

Cut off cleared 180 225 150 400 300 1255

The required percentage is

307 = 24.5% 24% 1255 Choice (3)

75. The sales of a company = The no .of units produced the closing stock. The sales of all the 5 companies in 2009 are as follows

Company P Q R S T

Sales 10515 14310 9225 7755 11135 Choice (2)

Hence S had the least sales in 2009.

76. The sales of R in 2008 = 9000 675 = 8325 The sales of R in 2009 = 10000 775 = 9225 R had lower sales in 2008. Choice (1)

Solutions for question 77 and 78: 77.

Solutions for questions 84 and 85: 84. The yield return of R in the years are as follows:

Family: Kapoor Khanna Kirsten Kumble Khan Kittu Kala

Total income (in `) 147000 105000 168000 140000 165000 120000 196000

Expenses + Overhead (In `) 12000 13500 17750 16750 19000 17450 21375 923175

Savings (in `) 135000 91500 150250 123250 146000 102550 174625

2003 2004 2005 2006 2007 2008

Yield Return 1557.14 1574.1 1884.6 2021.7 1595.2 1525

Hence the highest yield return is in 2021 Choice (1)


85. The yield return for Q in the years are as follows

The total savings made by all the families was `923175 Choice (2)
78. The increase in income of the Khan family is `3300 The decrease in expenses is `570 The increase in saving is 3300 + 570 = `3870 Choice (3) Solution for question 79: 79. The healthy drinks are S are X The other drinks are unhealthy. Hence the required ratio is 2 : 8 = 1 : 4 Solutions for 80 and 81: 80. The difference in the number of students studying in government schools in all the states in 2008 are as follows.

2003 2004 2005 2006 2007 2008

Yield Return 1676.5 2204.5 2633.3 2000 2720 4105.3

Hence the highest percentage increase is in 2008 Choice (4) Choice (3)
Solutions for questions 86 and 87: 86. From the table we can easily observe that the average marks are the highest for , X and X. Hence these classes would satisfy the statement "the higher the average marks, the higher are the number of students". Choice (3) 87. The statement the lower the number of students, the higher the average marks can be verified through the options. Classes and have higher number of students, hence they do not satisfy the statement. The correct choice is (A) Choice (1) Solutions for question 88:

AP MP UP Karnataka Kerala Tamil Nadu

Difference 1800 1600 1300 1400 2200 5200

The maximum increase is for Tamil Nadu, Choice (4)


81. We can observe that the state which has consistent increase in the number of students from 2007 to 2009 is UP. Choice (2) Solutions for questions 82 and 83: 82. The median of the total number of students is 18 + 17 =17.5 2 Hence A, C and F have more number of students than the median. Choice (2) 83. The number of failed students in each section is given the table. Hence the number of failed students is the highest more in section C

88.

The number of students this year in the 6 states Number of students 13,21,000 17,46,000 13,90,000 19,14,000 12,88,000 10,88,000

AP UP MP Bihar Assam Orissa

Hence MP has the third highest number of students this year Choice (2)
Solutions for question 89: 89.

Section A B C D E F

No. of students failed 6 8 12 7 3 10 Choice (3)

Given Lakshmi spends 20% of the revenue earned from each investment to maintain her house. So let us calculate the revenue for each business in which she invested. X 16.2 16.96 Y 14.5 14.72 Z 12.9 13.2

Investment Revenue

Hence the maximum profit is obtained from X Choice (1)

Solutions for questions 90 and 91: 90. The income and expenditure for the four regions in 2007 are as follows.

50 32 100 = 35 (approximately) 50

Choice (3)

Solutions for questions 5 and 6: 5.

Income North South East West 33.8 33.8 31.9 40.1

Expenditure 34.5 35.2 32.7 41.3

Ratio 0.98 0.96 0.975 0.971

Hence the required ratio is the highest for the North region. Choice (1)
91. The states in which the per capita income increased by more than 5% are J and K, West Bengal, Gujarat and Maharashtra In the remaining states the per capita income did not increase by more than 5% Hence the required ratio is 1 : 1 Choice (2) BAR GRAPH Solutions for questions 1 to 4: 1.

100 4800 = 12,000 40 100 4200 = 10,000 Number of bears in the world = 42 The number of deers and wild bisons in South America are 6,000 (25% of 24,000) and 5,400 (30% of 18,000) respectively. Number of wolves in South America = 25,800 (4,800 + 6,000 + 5,400 + 4,200) = 5,400 total number of wolves in the Amazon forest = 75% of 5,400 = 4,050. Choice (B) Number of pythons in the world =

6.

Species Pythons Deers Wild Bisons Wolves Bears

Number in Amazon Forest 80% (4,800) = 3,840 70% (6,000) = 4,200 80% (5,400) = 4,320 75% (5,400) = 4,050 95% (4,200) = 3,990 Choice (B)

The percentage growth rate in 2007 over 2006 250 190 = 100 = 31.5% 190 Had the percentage growth from 2007 to 2008 been 31.5%, the estimated revenue would have been 131.5 250 = 329 100 The required difference 329 305 = 25 (approximately) Choice (1) Let the number of people who used the company's products in Asia in 2003 be 100. The number of men and women who used the product in the different years are

Solutions for questions 7 to 9: 7.

The percentage increase is maximum in case of company R i.e.,


5400 4480 100 20.5% 4480

Choice (C)
8.

2.

Year 2003 2004 2005 2006 2007 2008 2009 2010 Men 60 63 66.15 69.5 73 76.65 80.5 84.5 Women 40 44 48.4 53.25 58.5 64.5 71 78 Total 100 162.5 the approximate percentage growth = 62 Choice (1) The percentage change in the gap between the revenues from the US and Asia in the different years are
9.

Since the cost of PC is same for all the companies market share of Q in 2009 4200 = 21.76% 5600 + 4200 + 5000 + 4500 Market share of Q in 2014 110% ( 4200 ) 100 = 28.37% = 110% (5600 + 4200 + 500 ) the difference in percentage points = 28.37 21.76 = 6.61. Choice (A) Looking at the options it is enough if we check for the market share of S in 2006 and 2007. Let the price per PC of A, B, C and D `x, `2x, `x and `2x respectively. Market share of S in 2006 2 5800 100 = 4200 + 3000 2 + 448 + 5800 2) 116 100 44% 42 + 60 + 44.8 + 116 Market share of S in 2007 108 54 2 = 100 = 100 43.5 + 28 2 + 48.5 + 54 2 256 = 42.2% Choice (A) =

3.

Year 2003 2004 2005 2006 2007 2008 2009 2010 Gap in 150 170 150 140 110 85 60 50 million USD Absolute percentage 13 12 6 21 22 30 17 change The absolute value of the percentage change in the growth rate was the highest in 2008-09. Choice (4)
4.

The growth rate in 2005 (over 2004) 135 90 = 100 = 50% 90 The growth rate in 2007 (over 2006) 250 190 = = 32% 190 The required percentage

10. Let us consider the selling prices of the four models in 2004 as 3k, 4k, 5k and 6k respectively. Selling prices in the years.

Models P Q R S

2004 3k 4k 5k 6k

2005 4.5k 6kl 7.5k 9k

2006 6k 8k 10k 12k

Sales revenue of Q in 2004 = 750 (4k) = 3000k Sales revenue of R in 2006 = 500(10k) = 5000k

Therefore the sales revenue of Q in 2004 by 2 5000k 3000k Choice (3) 100 = 66 % 3 3000k

Solutions for questions 11 to 13:

Car

2007 Production Sales 8000 20000 18000


4.

2008 Production 15000 21000 21000 Sales 10000 18000 20000

2009 Production 14000 25000 22000 Sales 9000 22000 16000

Alto Swift Estio


11. The total production of all three Cars in 2007 = 55000 2008 = 57000 2009 = 61000 The total sales of all three Cars in 2007 = 46000 2008 = 48000 2009 = 47000 The required ratio in 2007 = 1.19 2008 = 1.18 2009 = 1.29 Hence the ratio is the highest in 2009 12. The ratio of production to sales of Alto in 2007 = 1.625 2008 = 1.5 2009 = 1.55 The required ratio is the highest in 2007

13000 22000 20000

The student showed the highest percentage increase in 84 51 Choice (B) QA section i.e., 100 = 64.7% 51

5.

Section QA LR VA RC Choice (1) DI

Percentage change
56 34 100 > 50% 34 55.2 33.6 100 40% 55.2 36.4 24 100 > 50% 24 56 36 100 > 50% 36 62.5 52.4 100 16% 62.5

Choice (3)

13. The exports of Swift in 2007 = 2000 2008 = 3000 2009 = 3000 The ratio of exports to sales in 2007 = 0.1 2008 = 0.17 2009 = 0.14 The required ratio is the highest in 2008

Choice (D)
6.

The marks obtained by the student in the RC section is the highest in AIMCAT 5 Choice (B)

Solutions for question 7: 7.

Choice (2)
PIE CHART Solutions for questions 1 to 6:

Section Exam AIMCAT 1 AIMCAT 2 AIMCAT 3 AIMCAT 4 AIMCAT 5

QA 51 105 120 80 84

LR 69 30.8 45 64 42 QA 100

VA 60 35 60 32 91 LR 80

RC 45 56 54 64 70 VA 50

DI 75 53.2 81 80 63 RC 60

Total 300 280 360 320 350 DI 100

Maximum actual score


1.

Given the total number of students = 1000 Dancing = 45% = 450 Embroidery Classes = 5% = 50 Singing = 20% = 200 Karate = 15% = 150 Painting = 15% = 150 Now only Boys chose Karate. Hence a total 150 students in Karate are all boys. Only girls chose Embroidery classes. Hence a total 50 students in Embroidery are all girls. Also 80% of students in Singing are girls. Hence 160 students in Singing are girls and 40 are boys. Similarly 80% of students in Dancing are boys. Hence 360 are boys and 90 are girls, who are in dancing In Painting the ratio of boys to girls is 1 : 1 Let us tabulate the data. Boys 360 40 75 150 0 625 Girls 90 160 75 0 50 375 Total 450 200 150 150 50 1000

The required percentages =

60 100 = 120% 50 Choice (D)

2.

Maximum possible actual score in RC section = 60. The least difference occurs in the AIMCATs 2 and 4 i.e., 60 56 = 64 60 = 4 Choice (A) From the above tables, the given condition is satisfied in AIMCAT 3 and AIMCAT 5. Choice (B)

Dancing Singing Painting Karate Embroidery Class Total

3.

If Painting & Singing are mixed then the ratio of boys to girls is 115 : 235 = 23 : 47 Choice (3)

Solutions for question 8: 8.

LINE GRAPH Solutions for questions 1 and 2: 1.

Given the ratio of the number of employees in central to state government jobs is 6 : 1. Let the central government jobs be 600 and the state government jobs be 100. The number of central government employees in A.P = 150. The number of state government employees in Kerala = 25. The required ratio is 150 : 25 = 6 : 1 Choice (1)

Profit on a normal day = 7000 6500 = `500 Profit when 300 units are sold = 10,500 9000 = `1500 1500 500 Required percentage = 100 = 200% 500 Choice (C) Cost when 200 units are produced = `6500 Cost when 350 units are produced `10,000 Additional cost /unit = 10,000 6500 3500 = = `23. 150 150 Choice (B)

2.

Solutions for 9 and 10: 9.

The number of employees in each department of P and Q. P 1750 10500 5250 Q 3780 3060 11160

HR Academic Operations

Solutions for questions 3 and 4:

Total number of employees in both companies in the HR department = 5530 5530 = 15.5% Hence the required percentage = 35500 Choice (2)
10. The number of Academic employees = 13560 The number of Management department employees = 21940 13560 The required ratio = = 0.618 0.62 21940 Choice (2) Pie Charts + Bar Charts Solutions for questions 1 and 2:

The energy consumption of Geyser in a week is 7 kWh and we know the family uses the Geyser for 2 hrs in a day. Hence for 14 hrs in a week the energy consumption is 7 kWh. Hence the energy consumption of a Geyser per day is 1 kWh. Now, energy consumption of Refrigerator in a week is 14 kWh and the family uses Refrigerator throughout the day. Hence, the energy consumption of Refrigeration per day is 2 kWh. Similarly the energy consumption for TV in a day is 2 kWh. The energy consumption for Washing machine in a day is 1 4 kWh and for Grinder is kWh 7 7
3.

Number of students in each discipline is as follows: Number of students Discipline Marketing Finance Operations Systems HR Number of students 3780 1260 1008 1512 252

(a) Energy consumed by TV for 3 days = 6 kWh. Energy consumed by Refrigerator for 3 days = 6 kWh. Hence Choice (1) is false. (b) Energy consumed by Geyser for 4 days = 4 kWh. Energy consumed by Grinder for 7 days = 1 kWh. Hence Choice (2) is false. (c) Energy consumed by Washing Machine in a week = 4 kWh. Energy consumed by Geyser for 2 weeks = 14 kWh. Hence Choice (3) is true. (d) Energy consumed by TV for 2 days = 4 kWh. Energy consumed by Washing machine for one week = 4 kWh Hence (d) is false Choice (3) The fixed cost increased by 25 %. Hence the new fixed 1 cost is `60 + (60) = `75. 4 Hence the increment in the total cost is 15 15 100 = 100 = 12.5% 4 120 60 + (0.35 30) 7 Choice (2)
DATA SUFFICIENCY

4.

Number of males and females in each discipline are as follows: Males 2079 819 1008 840 576 5322 Females 1701 441 1512 672 672 4758 Difference 378 378 504 168 144

Marketing Finance HR Systems Operations Total


1.

Solutions for questions 1 to 4: 1.

The total number of female students in the institute was less than the total number of male students by 5322 4758 100 = 10.6% Choice (3) 5322 The difference between the number of male and female students was the highest for HR. Choice (1)

2.

From A, as 60% of the newly joined employees were not managers, the remaining 40% of the newly joined employees were managers. It is given that 10 managers had newly joined. 40% = 10 100% = 25 Hence, A alone is sufficient. B gives no data, it is just an assumption. Choice (1) From A and the given condition, either Babu or David got the highest rank. Hence, A alone is not sufficient.

2.

10

From B and the given condition, either David or Amar can be the highest ranker. Hence, B alone is not sufficient. Combining A and B, David must get the highest rank. Choice (4)
3.

It is given that, 30% of the students are boys, which implies 70% of the students are girls. Also 10% of the girls are athletes. 10% (70%) = 7% of the students are female athletes. From A, 25% of the students are athletes Hence 25 7 = 18% of the students are boys who are athletes. So, A alone is sufficient. From B, Number of boys who are athletes = 120% of the girls who are athletes. As 7% of the students are girls who are athletes, 120% (7%) = 8.4% of the students are boys who are athletes. So, B alone is also sufficient. Choice (3) Clearly, A alone is not sufficient, as we do not know how many points the opponent scored. B alone is also not sufficient, as we do not know how many points team A scored. Combing A and B, If the score at the half time was say 0-25, then the match would have ended in a tie at 35-35. So, team A did not win. Had the score at half time been, say, 10-35, then in the end it would have been 45-35 and team A would have won. So, we cannot answer the question even after combining both the statements. Choice (5)
PPL

Again yz = 21 the different possibilities are 1 21 37 But we do not know if x , y, z are natural numbers or not. For eg if y = 9, we can get. 7 xy = 18 as 29 and yz = 21 as 9 3 There will be infinite possibilities like this, so statement A is not sufficient. Statement B above is also not sufficient as it gives information regarding x and z only and nothing about y. Combining both the statements, we can conclude that x = 6, y = 3 and z = 7. Thus z is the maximum.
2.

4.

Using statement A alone we have SEVEN = 19 and FIVE = 14, without knowing anything about the values of individual alphabets, we cannot answer the question. Using both the statements together we can conclude that 2 (N) + I + E = 7 N = 1 and I and E are 2and 3 or 3 and 2 So F + V = 14 5 9. F and V could be 4, 5 or 5,4. SEVEN = 19 If E = 2 and V = 5, we get S = 9 whereas if E 3 and V = 5, we get S = 7. So we cannot determine S uniquely. Thus the question cannot be answered even by combining both the statements. The number of days that Raju's dad goes to the Shiva 365 temple in a year is = 121 days. 3 The number of days that Raju's dad goes to the 365 Venkateshwara temple is = 91 days. 4 The number of days that Raju's dad goes to the Saibaba temple is = 52 days. 7 The number of days that Raju's dad goes to both the Shiva and the Venkateshwara temple is 12 = 30 days. The number of days that Raju's dad goes to both the Venkateshwara and the Saibaba temple is 28 = 13 days. The number of days that Raju's dad goes to both the Shiva and the Saibaba temple is = 17 days. 21 The number of days he goes to all the three temples is

3.

1.

Let the cost of the new car be `x. 2 x. 5 From statement , we cannot answer the question as neither the cost of the new car nor the cost of the old car is given. Statement alone is not sufficient. From statement , we only know regarding his personal saving but nothing about the cost of the cars. Statement alone is not sufficient. By combining both the statements, we have the following information. 2 Amount borrowed from his friend = 60% 5 x Therefore the cost of the old car = 40% (x) = = 6 32 x = x 5 5 25

365

365

365

365

2 x 5 Money withdrawn from personal savings account to meet the cost of the new car. 6 2 9 x x= x =x 25 5 25 9 x was what portion of his Now, it is not known that 25 personal savings balance. Thus the question cannot be answered even by combining both the statements. Choice (4) Money realised by selling the old car =
(Numbers) 1.

365 84 = 4 days.
Hence the number of days Raju's dad goes to exactly one temple is 121 + 91 + 52 30 13 17 4 = 204 days. Choice (1)
4.

A + B +C + C + D + E +E +F + G = A + B + C + D + E +F +G + C + E (1 + 2 + 3 + 4 + 5 + 6 + 7) + C +E Now 28 + C +E = 33. C + E = 5 C, E could be (1, 4), (4, 1) (2, 3) OR (3, 2)there are four possible ordered pairs of (C, E). Choice (3)

From statement A, xy = 18 the different possibilities are: 118 29 36

11

CASELET Solutions for questions 1 to 3:

Solutions for questions 8 to 12:

The trading pattern followed by each of the three traders is as follows Anand Buy Sell 10 a.m. 3 p.m. Bala Chandu Buy Sell Buy Sell 10 a.m., 3 p.m. 10 a.m., 3 p.m. 11 a.m., 11 a.m., 12 noon, 12 noon, 1 p.m., 2 p.m.
8.

The arrangement of the buildings according to the given conditions is Yellow Blue Indigo B E C

1 p.m., 2 p.m.

1. 2/3. 4. 5. 6.
1.

Height E B/D A C F

A F Orange Green

D Violet

As the direction of the price movement is not known, the profits of Bala and Chandu depends upon the prices at which they bought gold i.e., if they buy at lesser price than that bought by Anand, their profits would be more, if not, the profits of Anand would be more than that of the other two. Hence the answer cannot be determined. Choice (5) Anand buys the entire quantity at a single point of time, whereas each of the other persons buy once every hour. As the direction of movement of gold is not given, we cannot compare the returns of Anand with the other two persons. Bala: Bala buys the same quantity of gold every time, irrespective of the price. Chandu: Chandu spends the same amount every time, his buying depends on the price of gold at the time he buys. The more the price, the lesser quantity he buys. As his strategy is based on prices, whenever the prices are changing, Chandus returns will be more than that of Bala. But if there is no change in the price of gold the returns of Bala and Chandu would be equal. Hence no conclusion can be made. Choice (5)

9.

The colour of the building diagonally opposite to the yellow coloured building is Violet. Choice (4) The second tallest building is either B or D. Choice (5) The colour of the tallest building is Indigo. Choice (2)

2. 3.

Solutions for questions 4 to 7:

Stage As P, Q, S and T won at least one match, R and U lost all the three matches. As Q, S and T lost at least one match, P won all the three matches. In stage-, there are a total of 9 matches and so 9 wins. Q, S and T won two matches each. As P (the top team in stage-) did not play against U, P played matches against Q and R. The ninth match was between Q and U. So the nine matches that have taken place are as follows. Won P Q P Lost S T Q Won S T P Lost R R R Won S T Q Lost U U U

10. On a boom day, the price of gold keeps rising, hence it will be the least in the morning. Hence, Anand who bought all his holdings in the morning will get the maximum profit. Between the remaining two, Bala bought the same quantity at every time, i.e he bought the same quantity even at higher prices whereas Chandu spent the same amount. Hence, Chandu bought less quantity of gold when prices were high and more when prices were less. Hence, Chandus returns are more than that of Bala's. Bala will have the least returns. Choice (1)

Let the prices of gold at different timings be as follows. Time 10 a.m. Price a 11 a.m. b 12 noon 1 p.m. 2 p.m. 3 p.m. c d e f

Stage- As each team played a total of five matches, in stage , the matches take place between the following pairs of teams. P T, P U, Q R, Q S, T S and R U Given that, in stage-, three teams lost all the two matches. Given P lost both the matches in stage- Each of T and U won the two matches. R and S lost the two matches. Q also won two matches.
4.

T and U defeated P (the top team in stage-) Choice (2) Only Q, T and U won both their matches in stage-. Choice (4) S and U won exactly two matches in the event. Choice (5) Q and T won exactly four matches each in the event. Choice (5)

5.

6.

7.

We will look at the additional information given: The quantity bought by Anand at 10 a.m. is the same as the quantity he sold at 3 p.m. As it is given that Anand lost money, we can ignore the quantity bought/sold and can conclude that the price at 3 p.m. must be less than that at 10 a.m. a>f () Similarly the quantity of gold bought/sold by Emma in each instance is the same and it is given that Emma made a profit. Hence we can conclude that (c + f) > (a + d) () Also using similar logic in case of David, we conclude that (d + e + f) > (a + b + c) () It is given that the price increased from 2 p.m. to 3 p.m. e<f (V) It is given that price at 12 noon was lower than the opening price c < a (V) From (i) and () we can conclude that c > d (V) From (), () and (V) we conclude that e > b (V) Hence a > f > e > b and a > c > d a is the highest.

12

11. The price of gold was the highest at 10 a.m. Choice (1) 12. As d < c, choice (4) is also necessarily false. Choice (4) Solutions for questions 13 to 15: 13. The different possibilities in which they could have booked the rooms are as follows. Case : 101 A 102 C 103 D 104 B

Students who opted for Finance = a + e + d + g. (a + e + d + g) 50% = b + f + c. (1) Students who did not opt for HR = a + e + b. a+e+d+g=
5 (a + e + b). 4

(2)

Again number of students who opted for only Finance and Marketing was 331/3% of those who opted for all three Number of students who opted for only Marketing and HR = Number of students who opted for only Finance and Marketing. f=e=
1 g. 3

Case :
101 B 102 D 103 C 104 A

Since B booked an odd numbered room, we can conclude that as per case , B must have booked room number 101, in which case C would have booked room number 103. Choice (C)
14. It is given that two girls failed in the examination. Now we have six possibilities in which we can select the two girls who failed. They are as follows: Cases 1 2 3 4 5 6 Dolly Pass Pass Pass Fail Fail Fail Molly Pass Fail Fail Pass Pass Fail Polly Fail Pass Fail Pass Fail Pass Kelly Fail Fail Pass Fail Pass Pass

f = e = k. From equation (1), a + e + d + g = 2 (b+ f + c) a + k + d + 3k = 2 (b + c) + 2k a + d = 2(8k a) 2k. 3a + d = 14k (3) From equation (2), we get, 4d + 11k = 5b + a (4) Again number of students who opted for only Finance & HR, i.e., d was 50% of those who opted for only Finance i.e., a. d = 50% a. Substituting in equation (3), we get d = 2k and a = 4k. Substituting in equation (4) we get b = 3k. Now (a + b + c + d + e + f + g) = 15k 15k = 270 k = 18 Exactly two = d + e + f = 4k = 4 (18) = 72 students. Choice (B)
Solutions for questions 16 and 17:

It is given that books D and F was read by the same person, A and B was not read by the same person and F and C was not read by the same person. The different combinations in which the books were read are as follows:
A D F E V A C E H

Let us denote a true statement by T and a false statement by L (lie)


Cases Dolly Molly Polly Kelly 1 T L T T 2 T L T L 3 T L L T 4 L L L T 5 L T T L 6 L L T T

B C G H A D F G
V

B C E H A D F H B C E G A C G H

V B D F E

As exactly three of them were telling the truth, only in case it is so. Thus Molly was the person who was lying. Choice (B)
15.

B D F G

A C E G

B D F H

Finance a = 4k e=k g=3k d = 2k f = k c=k b = 3k

Marketing

16. As Akira read books E and G, the books that Akira read could be either A, C, E and G or B, C, E and G. In either case, we can conclude that Aroki did not read book C. Choice (D) 17. As books C and E, were not read by the same person, as in cases and V, books G and H were read by the same person. Choice (C)

HR n

Solutions for questions 18 to 25: 18. W5 and W7 are allotted a shift, one earlier than W6 and W3 and W9 are also allotted a shift earlier than W6. Again as W3 is allotted a shift lower than W2, if we allot the afternoon shift for W3 and W9; W5 and W7 being one shift earlier than W6, we will have four workers in the Afternoon shift if W6 is allotted the Evening shift. Thus the only shift that can be alloted to W6 is the Night shift. The following table gives the workers and the shift they were allotted to.

It is given that n = O g = 37.5% of (a + b + c) i.e., g =


3 (a + b + c) 8

Let (a + b + c) be 8k. g = 3k Students who did not opt for Finance = b + f + c

13

Morning Afternoon Evening Night

W2 W1, W3, W9 W5, W7 W4, W6

Case (3) Schumi Sebastian Sebastian

Mclaren Schumi Schumi

Day 1 / 2 / 3 Sebastian Mclaren Mclaren

Thus W8 can be allotted any shift other than the afternoon shift. Choice (B)
19. The following table lists down the matches and the corresponding players who led the team as captain and vice captain. Match Match 1 Match 2 Match 3 Match 4 Match 5 Captain B A B C D Vice Captain A/D C A/D A/B/D A/B/C

If Schumi beats Mclaren on all the three days, then Mclaren will come last all the three days (not possible). Choice (D)
25. (i)

Both Sashi and Govind work together. This implies Ryan and Mokambo will work together Choice (A)

Solutions for questions 26 to 28: 26. Let the runs scored by Bhajji be x

As D refused to lead the team as captain if A or B led the team as captain in the preceding match, we can conclude that D can be the captain of the team only in Match 5. Again with D as Captain in Match 5, A must have captained the side in Match 2 for A cannot be the captain in Match 4. Now with A as the captain in Match 2, C must have been the vice captain in that match. Thus C was the vice captain in Match 2. Choice (C)
20. For B: Person B Appliances Water Purifier Refrigerator AC Day Monday Tuesday Wednesday

Straight drive Pull shot Others Total x + 40 Pollard x + 40 5 Dumminy (0.6) (x+ 20) (0.15) (2x + 40) x + 20 x = 20 x Bhajji 4

x = 20 4 x = 80.
Given Straight drive Pull shot Others Total 24 120 60 10 30 100 20 80

Pollard Dumminy Bhajji

Maximum possible difference = 95 10 = 85 Choice (D)


27. Maximum runs scored by Bhajji through straight drive = 59. 59 2 Required percentage = 100 = 19 /3% 300 Choice (D) 28. Runs scored by Bhajji through others cannot be determined. Choice (D) Solutions for questions 29 to 41: 29. (1) (2) (3) (4)

For A or C: AC must be bought before the water purifier. Choice (B) 21. Number of matches played by Sachin is equal to that played by Mongia. Number of matches played by Dravid is equal to that played by Hussey. Since Sachin has played more matches than Dravid, the average runs must be less than 45. Choice (A) 22. Average (Ramesh) =

282 = 40.28. 7 301 = 43. Average (Sanjay) = 7 If the average score after the exclusion lies between 40.28 and 43, then the average of Ramesh will decrease while that of Sanjay will increase. Since, 92 is the only value lying in that range, so their score in the invalid question is 42. Choice (A)

All shoes are pens. Not all pens are pencils. All pens are chocolates. Not all chocolates are pens.

23. A wins B wins B wins C does not win. That implies both A and C do not win together. That means at most one of A or C wins. That further implies that D must win. Choice (A) 24. Case (1) Schumi Schumi Sebastian Case (2) Schumi Schumi Sebasian Day 1 / 2 / 3 Sebastian Sebastian Mclaren Day 1 / 2 / 3 Mclaren Sebastian Mclaren

Analyzing the options: (A) Combining (1) and (4), we get Some chocolates are not shoes. (B) Combining (1) and (3), we get that some shoes are chocolates. Choice (D)
30.
t

Rameshs house

4
5 t

Umeshs house

Mclaren Mclaren Schumi

Total time = t +

4 9 t = t 5 5

Sebastian Mclaren Schumi

9 t = (7 : 45 4 ) pm = 225 min. 5 t = 125 min

14

When Ramesh reaches Umeshs house, his watch was showing 4 p.m. + 125 mins = 6:05 p.m. Umeshs watch was showing 6:10 p.m. So, Rameshs watch is 5 mins slower than Umeshs watch. Choice (D)
31.

T1

T2
34.

Emmanuels rank in Physics must have been 3. Ben got the same rank in Mathematics and Chemistry. Remaining ranks of Ben is be 4, 4 and 12. Therefore Ben or Emmanuel did not get the 1st rank in Mathematics. Thus Adams rank in Mathematics was 1. Cathy got 3rd rank in Mathematics, therefore Ben got the 4th rank in Mathematics. Proceeding like this we can conclude that Cathy got the 1st rank and Adam got the 2nd rank in Physics Choice (B)

200 x x

300 x y

Refrigerators b a d c

Air Conditioners

24
x 300 x + y = y 200 x + y

20

Let

x =2 y

26 LCD TVs

400 2y = 300 y y = 100 x If >2 y

then y > 100 and x > 200. This is not possible so only one value exists Choice (A)
32.

W X Y Z

Swimming Running Cycling Walking Total 5 6 15 1 6 5 2 4 14 1 18

It is given that at least 40 families own both a Refrigerator as well as a Air conditioner. b + d is at least 40. We have to find the maximum value of a. a will be maximum when (b + d) is minimum. i.e., when (b + d) is 40. Now a+ c+ (b + d) + 24 + 20 + 26 = 120. a + c = 120 70 (b + d) a + c = 10. The maximum value of a is 10, when c is 0. Thus at most 10 families owes a refrigerator and a LCD TV but not an Air Conditioner Choice (A)
35. The lectures and the days on which they deliver the lectures are tabulated in the following figure.

In case of Z: A sum total of 18 is possible when two 6s and one 5 is there in addition to 1.

Lecturers W X Y Z Swimming Running Cycling Walking Total 2 5 2 6 15 1 6 5 2 4 3 14 6 6 1 5 18

Monday L1

Tuesday Wednesday Thursday L4 L2 L1 L3

The above table gives the ranks obtained by the four persons in the four events. In case of X, a sum total of 6 is possible only if 1 + (1 + 3 + 1) is there. He has to get rank 3 in cycling and rank 1 in each of running and walking. Choice (A)
33.

L4 can deliver the lecture either on Tuesday or on Wednesday. Now, if L4 delivers his lecture on Wednesday, then L3 cannot deliver his lecture on any of the given days. [Since L3 delivers a lecture only if L2 delivered a lecture on the preceding day and L3 and L4 do not deliver lectures on consecutive days.] Thus L4 delivers the lecture on Tuesday. Now, the only day on which L3 could have delivered the lecture was Thursday Choice (D)
36. The different ways in which the committee can be formed is as follows: 1. B2 B4 B5 G2 G3 2. B1 B4 B5 G1 G3 3. B1 B4 B5 G2 G3 4. B1 B3 B5 G1 G3 5. B1 B3 B5 G2 G3 6. B2 B3 B5 G2 G3 Therefore there are six ways in which the committee can be formed Choice (D) 37. In order to have the total machining time as minimum, none of the machines must be idle at anytime and the total time taken must be 10 hours. (i.e., higher of the total machining times in the two machines). Let us consider the answer options and check if it is possible. Option A

Name Adam Ben Cathy Dimitry Emmanuel Subjects Mathematics Physics Chemistry Biology Total 1 2 3/5 5/3 11 4 5 4 2 15 3 1 5/3 3/5 12 2 4 1 1 8 5 3 2 4 14

Since the sum of the ranks of Dimitry was 8 and he got the same rank in Chemistry and Biology, his ranks in Chemistry and Biology was 1. Therefore Dimitrys rank in Mathematics was 2 which was the same as Emmanuels rank in Chemistry.

15

M1 Duration 2 3 5 Total = 10 Option B M1 Duration 3 2 5 Total = 10 P1 P3 P2 (4) P3 P1 P2 (4)

M2 P1 P2 P3 (5)

Case 1 M9 belonged to Germany. UK Germany France Switzerland Turkey M6 M9 M2 M1 M7. M8 M10 M4 M3 M5 Case 2 M9 belonged to Turkey. UK M6 M8 Germany France Switzerland Turkey M5 M2 M1 M7 M10 M4 M3 M9

M2 P2 P1 P3 (5)

(As M5 did not belong to France or Switzerland) Thus M10 belonged to Germany Choice (C)
40. It is given that sum of the costs of the gifts bought by Sneha and Sushma was equal to the cost of the gift bought by Shikha. We have four possibilities which satisfies this. Sneha Sushma Shikha Sushmita Case 1. 800 1200 2000 2800 Case 2. 1200 800 2000 2800 Case 3. 800 2000 2800 1200 Case 4. 2000 800 2800 1200 Again the difference between the cost of Sushmas gift and Sushmitas gift was equal to the cost of Snehas gift. This is satisfied only in Case 3. Thus the cost of the gift bought was Shikha was `2800 and she bought a pair of shoes. Choice (D) 41. As per the conditions given the different ways in which the terms for the two contest can be selected as follows: Debate: PVQ PVT PVS PVR PVT PVRPVR PVS PVU PVU PVR Elocution:PURPUR PURPUT PUS PUSPUT PUT PQS PQT PUQ Debate: PVS PVT Elocution:PUQ PUQ Option A is false as can be seen in the following cases: Debate: PVR PVS PVT Elocution:PUQ PUQPVQ Option B is true. If V and U are in the same category it must be for Debate. We know that U being in debate implies R is not in elocution. Again since only one among S and T can be selected for a particular category, Q must be selected. Option C is true as can be seen in the following cases: Debate: PVR PVS PVT Elocution:PUQ PUQPVQ Thus only statement given in option A is false Choice (A) NETWORKS 1.

In case of option (C), if product P2 is machined in M1 before Product P1, since P2 takes 4 hours in M1, it can be done as follows: Duration 0 4 47 79 M1 P2 P1 P3 Duration 0 5 57 7 10 M2 P3 P2 P1 The total time taken is again 10 hours Choice (D)
38.

Football a d e g e f b

Cricket

Hockey

Those playing exactly 3 games = g Those playing exactly 1 game = a + b + c Those playing exactly 2 games = d + e + f Those playing at least 2 games = d + e + f + g It is given that d + e + f + g = 18 (1) and (a + b + c) + (d + e + f + g) = 30 (2). Therefore a + b + c = 12. Now (a + b + c) = 3 (g). or, 3g = 12 or, g = 4. Therefore d + e + f = 18 4 = 14. Thus the number of members playing exactly two games is 14 Choice (B)
39. We can list down the names of the countries and the athletes belonging to them as follows. Countries UK Germany France SwitzerlandTurkey 1. M6 M7 M8 2.

The cost incurred will be minimum when the distance travelled is the minimum. The distance travelled is minimum when he takes a bus going via A E D F G H. The minimum cost incurred by him = 5 + 8 + 4 (10) = `53 Choice (3) If the road connecting A to E is under repair, then to incur minimum cost, one must board a bus going via the route A D F G H. Since the total distance travelled along this route is the least. The cost incurred = 5 + 8 + 4 (12) = `61 Choice (3)
QBR (Miscellaneous)

2.

It is given that M6 and M8 belonged to UK where as M7 belonged to Turkey. Now M5 and M9 were from different countries and M9 did not belong to France or Switzerland. So M9 belonged to either Germany or Turkey. Now M1 and M3 belonged to the same country and so did M2 and M4. Let us consider two cases.

1.

Given that the number of people rightly reported is 275. This includes people under C3 and C4. C3+ C4 = 275 .......... (1)

16

Number of people wrongly reported = 450 275 = 175 C1+ C2 = 175 . (2) Given the number of infected people is 50% that of non-infected Number of infected people = 150 = C1 + C4 . (3) And the number of non-infected people = 300 = C1 + C3 . (4) Required difference is between C2 and C4, obtained by (2) (3) C2 C4 = 175 150 = 25 Choice (2)
DI (Miscellaneous) 1.

Solutions for questions 4 and 5: 4.

From the choices only option (a) indicates low per capita income and low happiness quotient. Choice (1) From the choices only option (b) indicates high happiness quotient and high per capita income. Choice (2)

5.

Solution for question 6: 6.

Given that for every `2 increase in the selling price per ball, the number of balls sold decreases by 20. If the selling price of each ball is increased k times, selling price = `59 + 2k. Profit per ball = (59 + 2k) 50 = 9 + 2k. Number of balls sold = 700 20k. Profit obtained = (9 + 2k) (700 +20k). Profit = 10 ((9 + 2k) (70 2k)) = 10 (630 18k + 140k 4k2) = 10 (630 (4k2 122k)) 2 2 61 61 = 10 630 2k 2 2
2 2 61 61 = 10 630 + 2k 2 2 61 61 Profit is maximum when 2k = 0 2k = 2 2 But since k is an integer, 2k must be an integer. 60 62 2k can be taken to be or 2 2 If 2k = 30, profit = 10 ((9 + 30) (70 30)) = 10 39 40 = `115600 If 2k = 31, profit = 10 ((9 + 31) (70 31)) = 10 40 39 = `115600 For maximum profit, Selling price = 59 + 2k = 59 + 30 = `89 or 59 + 31 = `90 When selling price is `89, balls sold = 700 300 = 400 When selling price is `90, balls sold = 700 320 = 380 Of the given choices, only (A) satisfies. Choice (1)

Written Rahul Ramya 5 5

WE 3 1

Interview 2 3

Essay writing 3 4

GD 4 3

The cumulative score of Rahul is 5 0.3 + 3 0.1 + 2 0.25 + 3 0.1 + 4 0.25= 3.6 The cumulative score of Ramya is 5 0.3 + 1 0.1 + 3 0.25 + 4 0.1 + 3 0.25 = 3.5 The required difference is 0.1 Choice (1)
Solutions for 7 and 8: 7.

The expected pay-out for Raju is 80 0.5 + 40 0.3 20 0.2 = 48

Choice (2)

8.

The expected pay out for Ramu is 80 0.5 + 60 0.3 20 0.2 = 54 After the change of probability the expected pay out for Ramu is 80 0.3 + 60 0.5 20 0.2 = 50 4 The required percentage decrease is 100 54 = 7.4% Choice (1)

Solutions for question 9: 9.

2.

Total yield from scheme = 0.25 (3) + 0.55(80) + 0.2(100) = 7.5 + 44 +20 = 56.5 Therefore the total yield from scheme II was also 56.5. Let the probability of the bearish market be p. The probability of the bullish market = 1 0.4 P = 0.6 P Now, p ( 10) + 0.4 (60) + (0.6 p) 100 = 5605 110 p = 60 + 24 56.5 p = 0.25 Choice (3) Scheme 1 Market conditions Bearish Steady Bullish

The hotel cost for Ramu = $600 The cost incurred for city tour = $40 The cost incurred for tour of the Hunters valley = $35 Hence the total cost incurred by Ramu = $675 Choice (3)

3.

Probability 0.2 0.45 0.35

Yield percentage 30 80 100

10. Given that there has to be a male in every group. Hence only three groups can be formed. Also given P, S are in same group and each group has atleast one JSE and one SSE. As both P and S are JSE. The team should have one SSE. Given R is in a group of 3 people Hence the three groups should have 3, 3 and 2 members in each. Now considering the condition one JSE and one SSE in each group we get the following possibilities.

Group 1: Group 2: Group 3:

(i) R XY ZPS WQ

(ii) RXY ZQ WPS

The yield from scheme = 0.2 ( 30) + 0.45 (80) + 0.35 (100) = 6 + 36 + 35 = 65 Increase in the total yield from scheme 1 65 56.5 Choice (4) = 100 = 15% 56.5

From the above possibilities we can conclude that X should definitely be a member of a group which has 3 people. Choice (1)

17

Solutions for questions 11 and 12: 11. Let the value of the number in column 'b' and row 'd' be 'x' and that in column 'b' and row 'b' be 'A'. 1 (A + 26 + x) Given x = 3 1 2x = A + 26 x = ( A + 26) 2 As we know the grid contains only integers Therefore x should be an integer. Hence A should be even. From the choices only 16 is possible Choice (4) 12. Given the numbers in column 'a' are squares of the prime numbers, starting with the first odd prime number. Hence the numbers should be 32, 52, 72. 112 and 132. Thus, the required sum is 32 + 52 + 72 +112 + 132 = 373 Choice (4) Solutions for questions 13 and 14:

17. By observing we can easily find that R and S have the same visibility index. Choice (3) Solutions for question 18: 18. The points of A = 1 30 + 2 20 + 2 10 + 1 5 = 95 The points of B = 2 30 + 1 20 + 1 10 + 3 5 = 105 The points of C = 3 30 + 2 20 + 1 10 + 1 5 = 115 The points of E = 1 30 + 2 20 + 1 10 = 80 The winner is C Choice (3) Quant SI CI 1.

Let the number of years after which his interest in scheme 3 will be more than his interest from scheme 2 be n. 15 Interest from scheme 2, 2 = (10000 ) n = 1500n. 100

Given total number of sarees = 400 The ratio of Kanchipattu, Benarasi and Mangalgiri sarees is 5:3:2 The number of Kanchipattu sarees = 200 The number of Benarasi sarees = 120 The number of Mangalgiri sarees = 80 Given on day 1 he sells 20% of the total which is 80 on day 2 he sells 200 and on day3 he sells 120. Also on each day he sells the sarees in the same ratio as he bought i.e 5 : 3 : 2 Day1 Day2 Day3 Total
rd

10 1 + Interest from scheme 3, 3= 10000 10000. 200 For n = 8, 3 = `11829 & 2 = `12,000 For n = 9, 3 = `14066 & 2 = `13,500 Choice (2) After 9 years, 3 > 2.
2.

2n

Let us consider the interests received by him from the four schemes across the year with `1000 invested in each scheme.
Scheme 1 Scheme 2 80 150 166.4 300 259.7 450 360 600 469 750 587 900 714 1050 851 1200 Scheme 3 Scheme 4 102.5 80 216 166.4 340 259.7 477 360 629 469 796 587 980 714 1183 851

K 40 100 60 200

B 24 60 36 120

M 16 40 24 80

Total 80 200 120 400

13. On the 3 day he sold Benarasi saree at `480. Total amount received by him on the 3rd day is 60 350 + 36 480 +24 375 = `47280 Choice (2) 14. 25% of the total number of sarees were slightly damaged.

Year 1 Year 2 Year 3 Year 4 Year 5 Year 6 Year 7 Year 8

1 (400) = 100 4

The ratio of the damaged sarees of each type is 5 : 3 : 2. Hence the number of damaged Kanchipattu sarees = 50 The number of the damaged Benarasi sarees = 30 The number of the damaged Mangalgiri sarees = 20 He sold all the damaged sarees at 20% loss. Total amount = 50 280 + 30 320 + 20 300 = `29600 Choice (3)
Solutions for question 15: 15. The population of China in 2009 = 1.6 billiion As it increases by 12% per annum it becomes 5.56 billion in 2020 The population of China in 2020 is 15% of the total population 5.56 Hence the total population is = 37 billion 0.15 Choice (2) Solutions for questions 16 and 17: 16. The graph gives visibility index of 26 people. The visibility index of 14 people are more than U. 14 Hence the required ratio is 100 = 53.8 54% 26 Choice (2)

In scheme 4, amount at the end of the year = 1000 (1.2) = 1200. Amt. remaining after paying the administrative charges = 0.9 (1200) = 1080 Amt. at the end of the second year = 1080(1.2) = 1296 Amt. remaining after paying the administrative charges = 1296 (0.9) = 1166.4 This scheme is similar to scheme 1. Therefore scheme 2 produces the maximum interest at the end of 8 years. Choice (2)
LA (Venn Diagram) 1.

Let the number of students who applied for all three examinations be x. The number of students who applied for at least 2 of the 3 examinations = 36 2x
CAT 9x x 15x 12x XAT FMS

It is given that 25% (36 2x) = x 36 2x = 4x x=6 So the completed venn diagram will be as follows.

18

CAT 18 9 3 6 11 9 6

FMS

XAT

Number of students in the class = 18 + 3 + 6 + 9 + 6 + 11 = 62 Choice (3)


Solutions for questions 2 and 3:

We have the following information. The Hindu was read by 64 families. A + d + e = 55 (1) The times of India was read by 48 families. B + d + f = (2) The Telegraph was read by 45 families. C + e + f = 36 (3) Adding equations (1), (2) and (3), we get A + B + C + 2 (d + e+ f) = 130 Again A + B + C + d + e + f = 90 d + e + f = 40 A + B + C = 90 40 = 50 Therefore exactly one newspaper was read by 50 families. Choice (3)
Solutions for questions 5 and 6:

Speak in Hindi (480) e= 240 d = 120 t = 60 40 C = 180

Speak in English (500) B

Own a car (400) As per the data provided in the question, A + e + d = 360 (1) B + e + f = 380 (2) C + d + f = 280 (3) d + c = 24 (4) (3) (4) gives f = 40 Again d + 120 = 180 d = 60 c = 180 From (1), A + e = 300 and from (2), e +B = 340. Now A + e + e + B = 400 [ the total number of persons = 800] A + e + e + B = A + e + B + e = 300 + 340. e = 240 A = 60 and B = 100
2.

Given GT = 300 1 GT 300 a + d + f + g = 60 2 H S b + d + e + g =120 3 b c + e + f + g = 180 4 d a a+d=f+g5 g e f a=f=06 c + f = 30 7 c Equations 5 and 6 d = g n C Equations 6 and 7 c =30 Using GT formula a + b + c + d + e + f + g + n = 300 0 + (b + d + e + g) + 30 + 0 + n = 300 n = 60 Using equations 2, 5 & 6, we get d = g = 30.
5.

g 30 100 = 100 = 10 % GT 300 Choice (2)

6.

From 4 and c = 30, we get e = 120 Substituting in 2, Eqn. we get b = 30 Therefore b + n = 90. Choice (4)

7.

The number of persons who can speak in both Hindi and English = 240 + 120 = 360 360 The required percentage = 100 = 72% 500 Choice (3) The proportion of people in the locality who do not own B+e+ A +d+C a car or cannot speak in English = 800 640 = 0.8 Choice (4) = 800 A + B + C + D +E +F = 120 (21 + 9) = 90 The Hindu The Times of India d e 9 f C The Telegraph B

3.

Given, of the 300 students, 70 choose MS 230 Hence 230 choose MBA. Given g = 20 1 M F n=0 b d a + d + f + g = 100 2 a g b + e + d + g = 150 3 e f d = 2f = g d = 20, f = 10 4 c From 1, 3 and 4, we get n=0 H b + e = 110 GT = a + b + c + d + e + f + g + n 230 = 100 + b + e + c c = 20 Choice (3) Given the number of children who buy T & J (A) = 16 The number of children who buy C & H (B) = 26 The number of children who buy B & B (C) = 34 We know A + B + C = Ex1 + 2Ex2 + 3Ex3 Where Ex1, Ex2 and Ex3 denote the number of children buying exactly one, exactly two and exactly three toys respectively Given every child buys exactly 2 toys Hence Ex1 = Ex3 = 0 76 = 2Ex2 Ex2 = 38 Hence there are 38 children who visited the shop Choice (4)

8.

4.

21

Solutions for questions 9 and 10:

Given P = 24 (1) Q = 36 (2)

19

R = 29 .. (3) P a e b Q S = 25 .. (4) e + l + o + n = 12 .. (5) f + l + k + o = 18 (6) R f l g g + m + l + o = 16 .......... (7) c K = 8 and l + o = 6 .. (8) k o m h From (5) & (8) we get i j n d S e+n=6 P From (1) & (6) we get a+e+f+l+k+o+j+n= 24 a+e+n+j=6 but e + n = 6 a = j = 0 As the people who like S also like R. Hence d = i = n = j = 0. h + m + k + o = 25 Therefore e = 6 and b = 4.[As b + (g + l + m + o)= 36, where (g + l + m + o) = 16] From (6) & (8) f = 4 From (7) & (8) g + m = 10 From (3) c + h + g +m + l + o + f + k = 29 c + g + l + f + 25 = 29 c=g=l=0 Therefore h = 1, m = 10 and o =6
9.

Therefore the difference between the highest marks obtained is 20. Choice (1)
4.

The least score is obtained when the person attempts the following 3 papers. Paper 1 4 0 60 Paper 2 2 2 10 Paper 4 4 0 80 Choice (2)

Correct Wrong Score

Total score = 150


Solutions for questions 5 and 6:

The final arrangement of the persons in the 9 seater van was as follows:
1 2 3

U
4

X
5

Q
6

W
7

T
8

R
9

The number of people who like only Q = 14 Choice (4)


5.

10. The number of people who like all 4 movies is 6 Choice (2) LA (Miscellaneous) 1.

After the given swapping the final arrangement will be as follows:


1 2 3

Ranking of the stores and the total net scores for the pizza stores are as follows. Ranking as per price. 3 6 1 2 5 3 Ranking as per delivery Total net score time 3 3(0.7) +3(0.3) = 3.0 2 6 (0.7) +(0.3) = 4.8 1 1(0.7) + 1 (0.3) = 1.0 6 2(0.7) + 6(0.3) = 3.2 3 5(0.7) + 3 (0.3) = 4.4 5 3(0.7 + 5(0.3) = 3.6

W
4

T
5

Q
6

S
7

X
8

U
9

Stores A B C D E F

Therefore X will be seated beside U


6.

After the given swappings the final arrangement is as follows:


1 2 3

Therefore pizza store D got the third lowest net score. Choice (2)
Solutions for Questions 2 to 4: 2.

X
4

V
5

Q
6

S
7

U
8

R
9

The least score obtained by a person in the four papers can be obtained as follows. Paper 1 Paper 2 4 3 0 1 60 45 Paper 3 1 3 20 Paper 4 4 0 80

Correct Wrong Score

From the choices only "W is in the 9th seat" is correct. Choice (2)
Solutions for questions 7 to 9: 7.

Therefore the minimum net score that the person can get is 165. Choice (4)
3.

Correct Wrong Score

Paper 1 2 2 20

Paper 3 4 0 100 Total score = 120

Correct Wrong Score

Paper 2 4 0 110

Paper 4 1 2 30 Total score = 140

The proportion of residents who prefer watching movies PA is 0.65 The proportion of residents who prefer surfing the net, PB is 0.68 The proportion of residents who prefer doing both, PAB is 0.61 The proportion of residents who prefer at least one between watching movies or surfing net is PA + PB PAB = 0.65 + 0.68 0.61 = 0.72 Proportion of residents who neither watch movies nor surf net is 1 0.72 = 0.28 Let the population of A be 3k. Population of B is 5k, that of C is 3k and that of D is 4k.

8.

20

No. of residents who prefer chatting with friends in A is 0.36 3k = 1.08k in B is 0.45 5k = 2.25k in C is 0.32 3k = 0.96k in D is 0.25 4k = 1.00k Highest number is in B Choice (2)
9.

Hence the probability of Rajini taking something home


4

is

C1 2 = [since all the baskets are equally likely to C1 3


Choice (2)

get selected by Rajini]


Solutions for question 14:

No. of residents who prefer chatting with friends was calculated in the previous question. The average number of residents who prefer surfing net 1.9k + 2.75k + 2.04k + 3k = 2.4275k = 4 2 colonies have more than the average number. Choice (3)

10. The different ways in which oil can be transferred from tank B to tank H are 1. B E A F D C G H 2. B E A F G H 3. B E C A F G H 4. B E C G H 5. B E D A F G H 6. B E D C A F G H 7. B E D C G HS Thus there are seven possibilities in all. Choices (c) 11. Based on the conditions given in the question, we get the following possibilities.

14. Let us consider P = 1. When there is one goat and one tiger then the tiger eats the goat and gets transformed into a goat and stays happily in the forest. Now if P = 2 when there are two tigers. Now, if one of the tigers eat the goat then it gets transformed into a goat and then the second tiger would kill it. Hence when two tigers are there they would not kill the goat. Let P = 3. When three tigers are there. One of the three tigers kills the goat and becomes a goat. Hence the remaining tigers would not kill the goat. Hence when the tigers are odd numbered then they would kill the goat, else the goat is not eaten by any tiger. Choice (2) Solutions for questions 15 and 16:

Given a + g = 12 b+g=8 T a g b

GT P

Arun Varun Kiranmala 1. India-Day-to-Day India-Every-day India-These-days 2. India-These-Days India-Day-to-Day India-Every-day We can conclude that Varun did not subscribe to India-These Days. Choice (3)
12. If Kiranmala did not subscribe for India-These Days, then Varun subscribed for India-Day to Day. Choice (1) Solutions for question 13: 13. Given out of the six baskets four baskets have either gold or silver.

n
15. g = 4 The number of days that Ram learnt an instrument is a + b + g = 16 Choice (4) 16. Given a = 6 Hence g = 6 and b = 2 He learnt an instrument on a + b + g i.e 14 days in all. Choice (3) 17. From the given pattern we can understand that it is a cyclic pattern. Hence the input is repeated in every 7th step. Therefore step 28 would be the same as the input Choice (4)

LA (Circular Arrangements) 1.

The sitting arrangement was as follows

B A/H F F/H

B A

H/A

H/F

G E
2.

G E

Therefore from the above possibilities we can conclude that H is sitting opposite to C or D. From option (a) T R P W Q

Choice (4)

U cannot see P, W and Q, so option (a) is not correct From option (b) P T W Q S

U V Y X

V Y X U

21

All the conditions are satisfied, so (b) can be the answer From option (c) S T P W Q

Hence we can say that the Mumbai co-ordinator is opposite the Delhi co-ordinator. Choice (1)
Solutions for question 4: 4.

Given R and T sit together. We can arrange them in 2 ways. P and S do not sit together, so we can arrange them in 6 ways.

R U

X Y T cannot see y, so option (c) is not correct From option (d) P T Q W S

T/R R/T S S

V Y R

P R/T

T/R

P R/T

T/R

Bring the line below the diagram U cannot see P, so Choice (4) is not correct Choice (2)
Solutions for questions 3:

Using the first clue we can draw the figure as follows:

H T/R R/T
We can arrange the remaining two employees Q & U in 2 ways. Therefore, the total number of ways = 2 6 2 = 24 ways

M B
Using the 2nd clue we get,

Alternate solution:

H D

Considering R and T as a single unit, we get 3 units (R, T), Q and U which can be arranged around the table (3 1)! 2! ways [2! since RT can be arranged among the themselves] Now the remaining 2 persons can be placed in 2 of the 3 positions in 3C2 2! ways. Therefore the total number of arrangements = (3 1)! Choice (B) (2!) (3C2) 2! = 24 ways.

M B
LA (Distribution) Solutions for questions 1 to 3:

From the given data we can conclude that the twelve persons were living in the building as follows. Key board Singer Guitarist Guitarist Singer Instrumentalist player Q Floor 7 C R D P A Floor 12

Guitarist Singer Instrumentalist Drummer Singer Drummer F Floor 1 S Floor 2 B Floor 3 E/B T G/E

Floor 4 Floor 5 Floor 6

Floor 8 Floor 9 Floor 10 Floor11

22

1.

The four singers were S, T, C and P. Therefore only one male singer was there in the band. Choice (1) Above the floor in which G lived there were 6 floors or 8 floors. Choice (4) S lived in the second floor. Choice (2)
6.

on November 15th, 1998.and the one who got married on January 3rd, 1999. And it was 49 days. Choice (1) Based on the conditions given, the groups are as follows: Jalan, Kokila and Kadambar Jagan, Kavya and Kavita, Jeevan, Kavya and Kekul Therefore Jagan is in the same group as Kavya and Kavita Choice (1) Given B went to college on Thursday and did not teach Physics. As A, B and C did not teach Physics, we can conclude that D taught Physics. As A and C went to college on consecutive days it can be either on Monday and Tuesday, or Tuesday and Wednesday. They cannot go to college on Tuesday and Wednesday as Physics is taught after Chemistry. Hence A and C go to college on Monday and Tuesday respectively. Biology is taught on Monday. Choice (1)

2.

3.

Solutions for questions 4 and 5: 4.

From the given data, the only possibility is 1993 November 5th 1994 1995 1996 February 29th 1997 April 23rd, October 15th 1998 November 15th 1999 January 3rd. Both the couples who got married in the same month got married in the month of November. Choice (3) The least difference between the marriage dates of any two couples was between the couple who got married

7.

5.

Solutions for questions 8 and 9:

Given Team I scored maximum number of points, 364 is at the 15th place and Team A got 361 points. Given the sum of the points scored by teams at (13 + 14 + 15) is 1046. 364 + 361 + x = 1046 x = 321 Hence Team A is in the 14th place. As team C got 218 points and is in the 10th place and Team O got 251 points. Hence Team O should be placed between 11 -13. Given the ascending order of teams according to their points is O J F. Hence Team O is in the 11th place, team J is in the 12th place and Team F is in the 13th place with 321 points. Now we know the points of the teams in the 10th and the 11th place. Hence the points of the team in the 12th place is 284 points (from 10 + 11 + 12 = 753). As Team N and team B got 108 and 165 points respectively less than team F. Team N got 213 points and Team B got 156 points. The clue 7 + 8 + 9 = 590 implies that we can calculate the points of the third team in the above group. [We know N is one of the teams in the group] Hence 213 + 182 + x = 590. x = 195 points. Hence Team N is in the 9th place. The clue Team E got 18 points less than Team N, implies that team E is in the 8th place. From the clue 4 + 5 + 6 = 412, we know B is one of teams in the group. Hence 116 + 156 + x = 412 x = 140 Hence Team B is in the 6th place. The clue Team H got 4 points more than Team M, implies that Team K is in the 5th place. From the clue 1 + 2 + 3 = 302, we get 96 + x + 4 + x = 302 x = 101 Hence Team M got 101 points and is in the 2nd place. Therefore Team H got 105 points and is in the 3rd place. The final arrangement is as follows. Team Points
8. 9.

1st L 96

2nd M 101

3rd H 105

4th D 116

5th K 140

6th B 156

7th G 182

8th E 195

9th N 213

10th C 2182

11th O 251

12th J 284

13th F 321

14th A 361

15th I 364

The required difference is 284 140 = 144 Choice (2) The position of team E is 8th . Choice (2)

orders Fanta and V orders the same dish. Hence we know that V does not wear the Green shirt. Therefore S wear the Green shirt and he orders Pepsi. Let us tabulate the data. P Q R S T U V Shirt Orange Red Green Blue Drink Sprit Coke Pepsi Thums up

Solutions for questions 10 and 11:

Given P wears the Orange shirt and he orders Sprite and the person wearing the Green shirt orders Pepsi. From the clues, we know R wears the Red shirt, U wears the Blue shirt, T drinks Thumsup and Q drinks Coke. Hence R, Q, T and U neither wear Green nor order Pepsi. So the Green shirt is worn either by S or V. But from the clue, the person wearing Green shirt is the person who

23

Now from the last clue the person wearing the Violet shirt ordered Maaza. Hence V ordered Maaza and is wearing the Violet shirt.
10. S drinks Pepsi.

Choice (2)

11. V Violet shirt, Maaza is the correct combination. Choice (1) Solutions for questions 12 and 13:

It is given that the total work required to complete work 3 5 7 36 in A2 = N D (ND) + N D = 4 6 8 24 This will be true for infinite values of N. Thus a unique value of N cannot be determined from the given information. Choice (4)
Line + Bar graph Solutions for questions 1 to 4:

From the first and the last clue we get the following arrangement.

1.

O SA

L SL

From the third clue we know that the cricketer from India is at the extreme left end of the row and he plays for RCB. Also P is to the immediate right of him who plays for DC.

The number of accidents caused because of two 28 75000 = 21000. wheelers is 100 The accident severity index for two wheelers is 40. i.e. for every 100 accidents, 40 persons are killed. So for 21000 accidents 210 40 = 8400 persons are killed. Choice (3)

P IND RCB DC
nd

O SA

L SL

2.

Total accidents Trucks Bus Car Two wheelers Others 19500 13500 12000 21000 9000

Persons killed 6630 4050 4200 8400 4050

Persons injured 12870 9450 7800 12600 4950

x 0.52 0.43 0.54 0.67 0.82

Now from the 2 clue we know that N is to the immediate left to the player who plays for KKR. From the above consideration we get the final arrangement as

M IND, RCB

P N AUS, ENG, DC MI

O SA KKR

L SL DD

12. M plays for RCB and comes from India.

Choice (1)
13. The player from Australia plays for DC. Solution for questions 14 and 15: 14. Given X gave the presentation before T, R gave the presentation before V but after U, also W gave the presentation after P and S but before U and X. So, Q, U, X, T, R and V (need not be in the same order) conducted the seminars after W. Hence W should be giving his presentation in group 1 with P and S. Choice (1) 15. From the choices we can say that X, U and Q can be the first person to give the presentation in 2nd group but V cannot be the person. Choice (4) 16. Given R attends the Physics tuition, Q attends the Maths tuition. Also P and U same tuition, T and V attend same tuition and S does not attend the same tuition as Q. Hence S should attend a tuition with at least 2 students in it. i.e. S should either attends Physics or Chemistry as there should be at least 2 students in each tuition. Q should attend a tuition to which 3 people go. Choice (4) Quant ERPV 1. 4.

x = the ratio of the persons killed to the persons injured. The required ratio is the highest for other types of accidents. Choice (2)
3.

Choice (4)

The number of people who got injured by car accidents was 7800 Choice (2) The number of persons killed in truck is 6630 and number of persons injured in other type of accidents is 4950. The required difference is 1680 Choice (4)
DI (Distribution)

Solutions for questions 1 and 2:

As policies mature in between 1997 and 2002 and a policy matures on Feb 29th, the policy should mature on 29th Feb 2000. Now the policy which matures on Jan 10th matured after Feb 29th. Hence the policy on Jan 10th can be in 1997 or in 2002. But from one of the clues the policy on May 21st is the last and the Sep 17th policy is immediately before the May 21st policy. Hence the policies on Sep 17th and Jan 10th should mature the same year. The policy on August 8th is before Feb 29th. Hence August 8th policy is for either 98 or 99. The final arrangement is as follows. 3rd 4th 5th 6th th 29 Feb 24th Aug' 8th Aug 10th Jan 17th Sep May 21st 2000 1997 1998/1999 2001 2001 2002
1.

1st

2nd

The ratio of the floor areas of A1 and A2 is 1 : 4 Now the total work required to complete the work in 1 9 N 5 A1 = N (D) + D = 24 ND, where D = no. of 6 4 6 hours The total work required to complete the work in A 9 ND must be 4 . 24

The third matured policy is on 29th Feb 2000 Choice (1) In the year 2001, Atul receives money from two policies. Choice (2)

2.

24

LA(Linear Arrangement) Solutions for questions 1 to 3:

Given S is in 3rd place from the left end and the positions of F, Q, G are also given.

S UK

As two of F, Q, G are from USA and there is atleast one person between any two friends from USA, we can say that F and G are from USA. Also the extreme ends are occupied by friends from USA.

USA

F S UK USA

G USA

USA

USA

Now the friends from UK are separated by atleast four friends. So the other friend from UK can come either in the 2nd or the 4th position from the right. But from one of the clues P is from Australia and is in between and J. Hence we can get it as.

USA

Aus

F S Q UK USA Aus

G USA UK

P /J J/ USA Aus USA

Now as R is adjacent to and the friends at extreme ends are of different gender, we can get the final arrangement as follows.

T H USA Aus
1.

S F Q UK USA Aus

G R USA UK

P J USA Aus USA


Now D got the 4th rank D gets less than C. But the condition that one pair should have same marks is not satisfied. Hence E should get the highest marks. So E > C > A Now D cannot have more marks than C as D should have marks less than 3 students, hence C > D. As the first and the last ranked students do not have same marks as any other student, B should get the same marks as 'C'. Hence all the conditions are satisfied and the final arrangement is E > C = B> D > A.
1. 2.

F, G, and J stay in the USA. Therefore four female friends stay in the USA. Choice (4) Three friends are in between the friends from the UK Choice (1) T is the only male friend from USA. Choice (2)

2.

3.

Solution for question 4 and 5:

Given from (i) and (iv), the arrangement would be

E got the highest marks. B and C got the same marks. The descending order is ECBDA.

Choice (4) Choice (2) Choice (1)

From the (iii) clue, U and V have 2 persons in between them. Hence we have only one possibility.[since there is only one person between Tarun and Qureshi]

3.

Solutions for questions 4 and 5:

U/V

V/U

From the (ii) clue we get the final arrangement as

S
4.

U/V

V/U

Given at least 2 movies were released before R and there is one movie released between S and T. Hence R cannot be released on the 3rd or the 4th week of the month. It can only be released on the last week. Therefore Q is released on the 4th week. As S and T have one movie released between them, P is released on the 2nd week. Hence the final order would be as S/T, P, T/S, Q, R.
4. 5.

Raju is 3 places away to the left of Qureshi. Choice (4) Pradip is to the immediate right of Shyam. Choice (3)

R is released last. Only one movie is released before P.


Line Graph + Table

Choice (3) Choice (1)

5.

LA (Sequencing) Solutions for question 1: Solutions for 1 to 3: 1.

From the clue, D got less than 3 other students hence D got the 4th rank. From the other clue E > C > A. Let us assume B got the highest marks. Then the order is B > E > C > A.

From the table we can identify that the profit/ton in 2000 is the highest for Steel. From the line graph it is clear that the production of Steel is the highest. Hence the profit of Steel should be the highest. Choice (4)

25

You might also like